Change order of integration to obtain equivalent integrals












2












$begingroup$


In the textbook Thermal physics, by Garg, Bansal and Ghosh [1], there exist two integrals on page 640, which are claimed to be equivalent (without demonstration) by a change in the order of integration.



begin{align}
I_1&=int_{0}^{infty}dv_1e^{-frac{mv_1^2}{2k_BT}}v_1^2int_{0}^{v_1}dv_2e^{-frac{mv_2^2}{2k_BT}}left(frac{3v_1^2+v_2^2}{3v_1}right)v_2^2\
I_2=&int_{0}^{infty}dv_1e^{-frac{mv_1^2}{2k_BT}}v_1^2int_{v_1}^{infty}dv_2e^{-frac{mv_2^2}{2k_BT}}left(frac{3v_2^2+v_1^2}{3v_2}right)v_2^2
end{align}



It is stated that an interchange of the order of integration will show them to be equivalent. To show this, it must be done in such a way that the limits over $v_1$ in $I_1$, change from $0$ to $infty$ into $v_1$ to $infty$.



However, when attempting this problem myself I notice two things. The change in limits on $I_1$ doesn't appear to yield an equivalent region to integrate over, and even if it did, the functions to be integrated are not the same. This indicates to me that there is some other trick at play.




So the question follows: how can I prove these two integrals are the
same, and what other technique should be employed, besides a change in
integration, in order to show it?




To combine the integrals into something I can change the order in, I have been relabeling the first $v_1$ as $x$, then writing something like this:



begin{align}
I_1&=int_{0}^{v_1}int_{0}^{infty}e^{-frac{mx^2}{2k_BT}}x^2e^{-frac{mv_2^2}{2k_BT}}left(frac{3v_1^2+v_2^2}{3v_1}right)v_2^2dxdv_2\
end{align}



I'm asking this question because I promised @Thorondor I'd show them to be equivalent in this question [2], but unfortunately I got stumped.



References:



[1] Garg S.C. Bansal R.M. Ghosh C.K. (2012) Thermal Physics: Kinetic Theory, Thermodynamics and Statistical Mechanics. Tata McGraw Hill Education, New Delhi, 638-641.



[2] https://physics.stackexchange.com/questions/448761/rigorous-derivation-of-the-mean-free-path-in-a-gas










share|cite|improve this question









$endgroup$

















    2












    $begingroup$


    In the textbook Thermal physics, by Garg, Bansal and Ghosh [1], there exist two integrals on page 640, which are claimed to be equivalent (without demonstration) by a change in the order of integration.



    begin{align}
    I_1&=int_{0}^{infty}dv_1e^{-frac{mv_1^2}{2k_BT}}v_1^2int_{0}^{v_1}dv_2e^{-frac{mv_2^2}{2k_BT}}left(frac{3v_1^2+v_2^2}{3v_1}right)v_2^2\
    I_2=&int_{0}^{infty}dv_1e^{-frac{mv_1^2}{2k_BT}}v_1^2int_{v_1}^{infty}dv_2e^{-frac{mv_2^2}{2k_BT}}left(frac{3v_2^2+v_1^2}{3v_2}right)v_2^2
    end{align}



    It is stated that an interchange of the order of integration will show them to be equivalent. To show this, it must be done in such a way that the limits over $v_1$ in $I_1$, change from $0$ to $infty$ into $v_1$ to $infty$.



    However, when attempting this problem myself I notice two things. The change in limits on $I_1$ doesn't appear to yield an equivalent region to integrate over, and even if it did, the functions to be integrated are not the same. This indicates to me that there is some other trick at play.




    So the question follows: how can I prove these two integrals are the
    same, and what other technique should be employed, besides a change in
    integration, in order to show it?




    To combine the integrals into something I can change the order in, I have been relabeling the first $v_1$ as $x$, then writing something like this:



    begin{align}
    I_1&=int_{0}^{v_1}int_{0}^{infty}e^{-frac{mx^2}{2k_BT}}x^2e^{-frac{mv_2^2}{2k_BT}}left(frac{3v_1^2+v_2^2}{3v_1}right)v_2^2dxdv_2\
    end{align}



    I'm asking this question because I promised @Thorondor I'd show them to be equivalent in this question [2], but unfortunately I got stumped.



    References:



    [1] Garg S.C. Bansal R.M. Ghosh C.K. (2012) Thermal Physics: Kinetic Theory, Thermodynamics and Statistical Mechanics. Tata McGraw Hill Education, New Delhi, 638-641.



    [2] https://physics.stackexchange.com/questions/448761/rigorous-derivation-of-the-mean-free-path-in-a-gas










    share|cite|improve this question









    $endgroup$















      2












      2








      2





      $begingroup$


      In the textbook Thermal physics, by Garg, Bansal and Ghosh [1], there exist two integrals on page 640, which are claimed to be equivalent (without demonstration) by a change in the order of integration.



      begin{align}
      I_1&=int_{0}^{infty}dv_1e^{-frac{mv_1^2}{2k_BT}}v_1^2int_{0}^{v_1}dv_2e^{-frac{mv_2^2}{2k_BT}}left(frac{3v_1^2+v_2^2}{3v_1}right)v_2^2\
      I_2=&int_{0}^{infty}dv_1e^{-frac{mv_1^2}{2k_BT}}v_1^2int_{v_1}^{infty}dv_2e^{-frac{mv_2^2}{2k_BT}}left(frac{3v_2^2+v_1^2}{3v_2}right)v_2^2
      end{align}



      It is stated that an interchange of the order of integration will show them to be equivalent. To show this, it must be done in such a way that the limits over $v_1$ in $I_1$, change from $0$ to $infty$ into $v_1$ to $infty$.



      However, when attempting this problem myself I notice two things. The change in limits on $I_1$ doesn't appear to yield an equivalent region to integrate over, and even if it did, the functions to be integrated are not the same. This indicates to me that there is some other trick at play.




      So the question follows: how can I prove these two integrals are the
      same, and what other technique should be employed, besides a change in
      integration, in order to show it?




      To combine the integrals into something I can change the order in, I have been relabeling the first $v_1$ as $x$, then writing something like this:



      begin{align}
      I_1&=int_{0}^{v_1}int_{0}^{infty}e^{-frac{mx^2}{2k_BT}}x^2e^{-frac{mv_2^2}{2k_BT}}left(frac{3v_1^2+v_2^2}{3v_1}right)v_2^2dxdv_2\
      end{align}



      I'm asking this question because I promised @Thorondor I'd show them to be equivalent in this question [2], but unfortunately I got stumped.



      References:



      [1] Garg S.C. Bansal R.M. Ghosh C.K. (2012) Thermal Physics: Kinetic Theory, Thermodynamics and Statistical Mechanics. Tata McGraw Hill Education, New Delhi, 638-641.



      [2] https://physics.stackexchange.com/questions/448761/rigorous-derivation-of-the-mean-free-path-in-a-gas










      share|cite|improve this question









      $endgroup$




      In the textbook Thermal physics, by Garg, Bansal and Ghosh [1], there exist two integrals on page 640, which are claimed to be equivalent (without demonstration) by a change in the order of integration.



      begin{align}
      I_1&=int_{0}^{infty}dv_1e^{-frac{mv_1^2}{2k_BT}}v_1^2int_{0}^{v_1}dv_2e^{-frac{mv_2^2}{2k_BT}}left(frac{3v_1^2+v_2^2}{3v_1}right)v_2^2\
      I_2=&int_{0}^{infty}dv_1e^{-frac{mv_1^2}{2k_BT}}v_1^2int_{v_1}^{infty}dv_2e^{-frac{mv_2^2}{2k_BT}}left(frac{3v_2^2+v_1^2}{3v_2}right)v_2^2
      end{align}



      It is stated that an interchange of the order of integration will show them to be equivalent. To show this, it must be done in such a way that the limits over $v_1$ in $I_1$, change from $0$ to $infty$ into $v_1$ to $infty$.



      However, when attempting this problem myself I notice two things. The change in limits on $I_1$ doesn't appear to yield an equivalent region to integrate over, and even if it did, the functions to be integrated are not the same. This indicates to me that there is some other trick at play.




      So the question follows: how can I prove these two integrals are the
      same, and what other technique should be employed, besides a change in
      integration, in order to show it?




      To combine the integrals into something I can change the order in, I have been relabeling the first $v_1$ as $x$, then writing something like this:



      begin{align}
      I_1&=int_{0}^{v_1}int_{0}^{infty}e^{-frac{mx^2}{2k_BT}}x^2e^{-frac{mv_2^2}{2k_BT}}left(frac{3v_1^2+v_2^2}{3v_1}right)v_2^2dxdv_2\
      end{align}



      I'm asking this question because I promised @Thorondor I'd show them to be equivalent in this question [2], but unfortunately I got stumped.



      References:



      [1] Garg S.C. Bansal R.M. Ghosh C.K. (2012) Thermal Physics: Kinetic Theory, Thermodynamics and Statistical Mechanics. Tata McGraw Hill Education, New Delhi, 638-641.



      [2] https://physics.stackexchange.com/questions/448761/rigorous-derivation-of-the-mean-free-path-in-a-gas







      integration physics






      share|cite|improve this question













      share|cite|improve this question











      share|cite|improve this question




      share|cite|improve this question










      asked Jan 2 at 5:14









      user400188user400188

      4291614




      4291614






















          1 Answer
          1






          active

          oldest

          votes


















          1












          $begingroup$

          Write the inner integral of $I_1$ as



          $$int_{0}^{infty}dv_2e^{-frac{mv_2^2}{2k_BT}}left(frac{3v_1^2+v_2^2}{3v_1}right)v_2^2 mathbf{1}_{{v_1 geqslant v_2}}$$



          where



          $$mathbf{1}_{{v_1 geqslant v_2}} = begin{cases}1, & v_1 geqslant v_2 \ 0 , & v_1 < v_2end{cases}$$



          Apply Fubini's theorem to interchange integrals noting that



          $$int_{0}^{infty}dv_1f(v_1,v_2)mathbf{1}_{{v_1 geqslant v_2}} = int_{v_2}^{infty}dv_1f(v_1,v_2) $$



          and then change notation $v_1 iff v_2$.






          share|cite|improve this answer









          $endgroup$













          • $begingroup$
            Would it be correct to say here, that we are integrating over different regions, but the areas of both regions are the same, justifying the change in notation? I'm just trying to verify if my understanding of the answer is correct.
            $endgroup$
            – user400188
            Jan 2 at 5:46






          • 1




            $begingroup$
            If you follow my steps you get $I_1 = int_{0}^{infty}dv_2e^{-frac{mv_2^2}{2k_BT}}v_2^2int_{v_2}^{infty}dv_1e^{-frac{mv_1^2}{2k_BT}}left(frac{3v_1^2+v_2^2}{3v_1}right)v_1^2$ before changing notation. I think you can see why this works just by looking at a graph of the original region of integration -- an unbounded triangle-like region. Now switching variable names requires no deep mathematical justification for a definite integral.
            $endgroup$
            – RRL
            Jan 2 at 5:55










          • $begingroup$
            Thanks @RRL, I drew the graphs earlier and wanted to confirm my suspicion.
            $endgroup$
            – user400188
            Jan 2 at 5:56












          Your Answer








          StackExchange.ready(function() {
          var channelOptions = {
          tags: "".split(" "),
          id: "69"
          };
          initTagRenderer("".split(" "), "".split(" "), channelOptions);

          StackExchange.using("externalEditor", function() {
          // Have to fire editor after snippets, if snippets enabled
          if (StackExchange.settings.snippets.snippetsEnabled) {
          StackExchange.using("snippets", function() {
          createEditor();
          });
          }
          else {
          createEditor();
          }
          });

          function createEditor() {
          StackExchange.prepareEditor({
          heartbeatType: 'answer',
          autoActivateHeartbeat: false,
          convertImagesToLinks: true,
          noModals: true,
          showLowRepImageUploadWarning: true,
          reputationToPostImages: 10,
          bindNavPrevention: true,
          postfix: "",
          imageUploader: {
          brandingHtml: "Powered by u003ca class="icon-imgur-white" href="https://imgur.com/"u003eu003c/au003e",
          contentPolicyHtml: "User contributions licensed under u003ca href="https://creativecommons.org/licenses/by-sa/3.0/"u003ecc by-sa 3.0 with attribution requiredu003c/au003e u003ca href="https://stackoverflow.com/legal/content-policy"u003e(content policy)u003c/au003e",
          allowUrls: true
          },
          noCode: true, onDemand: true,
          discardSelector: ".discard-answer"
          ,immediatelyShowMarkdownHelp:true
          });


          }
          });














          draft saved

          draft discarded


















          StackExchange.ready(
          function () {
          StackExchange.openid.initPostLogin('.new-post-login', 'https%3a%2f%2fmath.stackexchange.com%2fquestions%2f3059156%2fchange-order-of-integration-to-obtain-equivalent-integrals%23new-answer', 'question_page');
          }
          );

          Post as a guest















          Required, but never shown

























          1 Answer
          1






          active

          oldest

          votes








          1 Answer
          1






          active

          oldest

          votes









          active

          oldest

          votes






          active

          oldest

          votes









          1












          $begingroup$

          Write the inner integral of $I_1$ as



          $$int_{0}^{infty}dv_2e^{-frac{mv_2^2}{2k_BT}}left(frac{3v_1^2+v_2^2}{3v_1}right)v_2^2 mathbf{1}_{{v_1 geqslant v_2}}$$



          where



          $$mathbf{1}_{{v_1 geqslant v_2}} = begin{cases}1, & v_1 geqslant v_2 \ 0 , & v_1 < v_2end{cases}$$



          Apply Fubini's theorem to interchange integrals noting that



          $$int_{0}^{infty}dv_1f(v_1,v_2)mathbf{1}_{{v_1 geqslant v_2}} = int_{v_2}^{infty}dv_1f(v_1,v_2) $$



          and then change notation $v_1 iff v_2$.






          share|cite|improve this answer









          $endgroup$













          • $begingroup$
            Would it be correct to say here, that we are integrating over different regions, but the areas of both regions are the same, justifying the change in notation? I'm just trying to verify if my understanding of the answer is correct.
            $endgroup$
            – user400188
            Jan 2 at 5:46






          • 1




            $begingroup$
            If you follow my steps you get $I_1 = int_{0}^{infty}dv_2e^{-frac{mv_2^2}{2k_BT}}v_2^2int_{v_2}^{infty}dv_1e^{-frac{mv_1^2}{2k_BT}}left(frac{3v_1^2+v_2^2}{3v_1}right)v_1^2$ before changing notation. I think you can see why this works just by looking at a graph of the original region of integration -- an unbounded triangle-like region. Now switching variable names requires no deep mathematical justification for a definite integral.
            $endgroup$
            – RRL
            Jan 2 at 5:55










          • $begingroup$
            Thanks @RRL, I drew the graphs earlier and wanted to confirm my suspicion.
            $endgroup$
            – user400188
            Jan 2 at 5:56
















          1












          $begingroup$

          Write the inner integral of $I_1$ as



          $$int_{0}^{infty}dv_2e^{-frac{mv_2^2}{2k_BT}}left(frac{3v_1^2+v_2^2}{3v_1}right)v_2^2 mathbf{1}_{{v_1 geqslant v_2}}$$



          where



          $$mathbf{1}_{{v_1 geqslant v_2}} = begin{cases}1, & v_1 geqslant v_2 \ 0 , & v_1 < v_2end{cases}$$



          Apply Fubini's theorem to interchange integrals noting that



          $$int_{0}^{infty}dv_1f(v_1,v_2)mathbf{1}_{{v_1 geqslant v_2}} = int_{v_2}^{infty}dv_1f(v_1,v_2) $$



          and then change notation $v_1 iff v_2$.






          share|cite|improve this answer









          $endgroup$













          • $begingroup$
            Would it be correct to say here, that we are integrating over different regions, but the areas of both regions are the same, justifying the change in notation? I'm just trying to verify if my understanding of the answer is correct.
            $endgroup$
            – user400188
            Jan 2 at 5:46






          • 1




            $begingroup$
            If you follow my steps you get $I_1 = int_{0}^{infty}dv_2e^{-frac{mv_2^2}{2k_BT}}v_2^2int_{v_2}^{infty}dv_1e^{-frac{mv_1^2}{2k_BT}}left(frac{3v_1^2+v_2^2}{3v_1}right)v_1^2$ before changing notation. I think you can see why this works just by looking at a graph of the original region of integration -- an unbounded triangle-like region. Now switching variable names requires no deep mathematical justification for a definite integral.
            $endgroup$
            – RRL
            Jan 2 at 5:55










          • $begingroup$
            Thanks @RRL, I drew the graphs earlier and wanted to confirm my suspicion.
            $endgroup$
            – user400188
            Jan 2 at 5:56














          1












          1








          1





          $begingroup$

          Write the inner integral of $I_1$ as



          $$int_{0}^{infty}dv_2e^{-frac{mv_2^2}{2k_BT}}left(frac{3v_1^2+v_2^2}{3v_1}right)v_2^2 mathbf{1}_{{v_1 geqslant v_2}}$$



          where



          $$mathbf{1}_{{v_1 geqslant v_2}} = begin{cases}1, & v_1 geqslant v_2 \ 0 , & v_1 < v_2end{cases}$$



          Apply Fubini's theorem to interchange integrals noting that



          $$int_{0}^{infty}dv_1f(v_1,v_2)mathbf{1}_{{v_1 geqslant v_2}} = int_{v_2}^{infty}dv_1f(v_1,v_2) $$



          and then change notation $v_1 iff v_2$.






          share|cite|improve this answer









          $endgroup$



          Write the inner integral of $I_1$ as



          $$int_{0}^{infty}dv_2e^{-frac{mv_2^2}{2k_BT}}left(frac{3v_1^2+v_2^2}{3v_1}right)v_2^2 mathbf{1}_{{v_1 geqslant v_2}}$$



          where



          $$mathbf{1}_{{v_1 geqslant v_2}} = begin{cases}1, & v_1 geqslant v_2 \ 0 , & v_1 < v_2end{cases}$$



          Apply Fubini's theorem to interchange integrals noting that



          $$int_{0}^{infty}dv_1f(v_1,v_2)mathbf{1}_{{v_1 geqslant v_2}} = int_{v_2}^{infty}dv_1f(v_1,v_2) $$



          and then change notation $v_1 iff v_2$.







          share|cite|improve this answer












          share|cite|improve this answer



          share|cite|improve this answer










          answered Jan 2 at 5:37









          RRLRRL

          53.9k52675




          53.9k52675












          • $begingroup$
            Would it be correct to say here, that we are integrating over different regions, but the areas of both regions are the same, justifying the change in notation? I'm just trying to verify if my understanding of the answer is correct.
            $endgroup$
            – user400188
            Jan 2 at 5:46






          • 1




            $begingroup$
            If you follow my steps you get $I_1 = int_{0}^{infty}dv_2e^{-frac{mv_2^2}{2k_BT}}v_2^2int_{v_2}^{infty}dv_1e^{-frac{mv_1^2}{2k_BT}}left(frac{3v_1^2+v_2^2}{3v_1}right)v_1^2$ before changing notation. I think you can see why this works just by looking at a graph of the original region of integration -- an unbounded triangle-like region. Now switching variable names requires no deep mathematical justification for a definite integral.
            $endgroup$
            – RRL
            Jan 2 at 5:55










          • $begingroup$
            Thanks @RRL, I drew the graphs earlier and wanted to confirm my suspicion.
            $endgroup$
            – user400188
            Jan 2 at 5:56


















          • $begingroup$
            Would it be correct to say here, that we are integrating over different regions, but the areas of both regions are the same, justifying the change in notation? I'm just trying to verify if my understanding of the answer is correct.
            $endgroup$
            – user400188
            Jan 2 at 5:46






          • 1




            $begingroup$
            If you follow my steps you get $I_1 = int_{0}^{infty}dv_2e^{-frac{mv_2^2}{2k_BT}}v_2^2int_{v_2}^{infty}dv_1e^{-frac{mv_1^2}{2k_BT}}left(frac{3v_1^2+v_2^2}{3v_1}right)v_1^2$ before changing notation. I think you can see why this works just by looking at a graph of the original region of integration -- an unbounded triangle-like region. Now switching variable names requires no deep mathematical justification for a definite integral.
            $endgroup$
            – RRL
            Jan 2 at 5:55










          • $begingroup$
            Thanks @RRL, I drew the graphs earlier and wanted to confirm my suspicion.
            $endgroup$
            – user400188
            Jan 2 at 5:56
















          $begingroup$
          Would it be correct to say here, that we are integrating over different regions, but the areas of both regions are the same, justifying the change in notation? I'm just trying to verify if my understanding of the answer is correct.
          $endgroup$
          – user400188
          Jan 2 at 5:46




          $begingroup$
          Would it be correct to say here, that we are integrating over different regions, but the areas of both regions are the same, justifying the change in notation? I'm just trying to verify if my understanding of the answer is correct.
          $endgroup$
          – user400188
          Jan 2 at 5:46




          1




          1




          $begingroup$
          If you follow my steps you get $I_1 = int_{0}^{infty}dv_2e^{-frac{mv_2^2}{2k_BT}}v_2^2int_{v_2}^{infty}dv_1e^{-frac{mv_1^2}{2k_BT}}left(frac{3v_1^2+v_2^2}{3v_1}right)v_1^2$ before changing notation. I think you can see why this works just by looking at a graph of the original region of integration -- an unbounded triangle-like region. Now switching variable names requires no deep mathematical justification for a definite integral.
          $endgroup$
          – RRL
          Jan 2 at 5:55




          $begingroup$
          If you follow my steps you get $I_1 = int_{0}^{infty}dv_2e^{-frac{mv_2^2}{2k_BT}}v_2^2int_{v_2}^{infty}dv_1e^{-frac{mv_1^2}{2k_BT}}left(frac{3v_1^2+v_2^2}{3v_1}right)v_1^2$ before changing notation. I think you can see why this works just by looking at a graph of the original region of integration -- an unbounded triangle-like region. Now switching variable names requires no deep mathematical justification for a definite integral.
          $endgroup$
          – RRL
          Jan 2 at 5:55












          $begingroup$
          Thanks @RRL, I drew the graphs earlier and wanted to confirm my suspicion.
          $endgroup$
          – user400188
          Jan 2 at 5:56




          $begingroup$
          Thanks @RRL, I drew the graphs earlier and wanted to confirm my suspicion.
          $endgroup$
          – user400188
          Jan 2 at 5:56


















          draft saved

          draft discarded




















































          Thanks for contributing an answer to Mathematics Stack Exchange!


          • Please be sure to answer the question. Provide details and share your research!

          But avoid



          • Asking for help, clarification, or responding to other answers.

          • Making statements based on opinion; back them up with references or personal experience.


          Use MathJax to format equations. MathJax reference.


          To learn more, see our tips on writing great answers.




          draft saved


          draft discarded














          StackExchange.ready(
          function () {
          StackExchange.openid.initPostLogin('.new-post-login', 'https%3a%2f%2fmath.stackexchange.com%2fquestions%2f3059156%2fchange-order-of-integration-to-obtain-equivalent-integrals%23new-answer', 'question_page');
          }
          );

          Post as a guest















          Required, but never shown





















































          Required, but never shown














          Required, but never shown












          Required, but never shown







          Required, but never shown

































          Required, but never shown














          Required, but never shown












          Required, but never shown







          Required, but never shown







          Popular posts from this blog

          Biblatex bibliography style without URLs when DOI exists (in Overleaf with Zotero bibliography)

          ComboBox Display Member on multiple fields

          Is it possible to collect Nectar points via Trainline?